18
$\begingroup$

The Brown-Comenetz dualizing spectrum $I_{\mathbf{Q/Z}}$ is not detected by very many spectra: it is $BP, \mathbf{Z}, \mathbf{F}_2, X(n)$ for $n\geq 2$, and even $I_{\mathbf{Q/Z}}$-acyclic. However, if $X$ is any nontrivial finite spectrum, then $X\wedge I_{\mathbf{Q/Z}}$ is not contractible. This motivates a natural question: let $E$ be any spectrum such that $E\wedge I_{\mathbf{Q/Z}}$ is not contractible. Then, is it true that $\langle E\rangle \geq \langle X\rangle$ for some nontrivial finite spectrum $X$?

$\endgroup$
5
  • 1
    $\begingroup$ A silly comment, but X(1) is the sphere spectrum, so you probably don't want to include that in your list of examples of spectra that don't detect. $\endgroup$ Jun 3, 2018 at 8:29
  • $\begingroup$ You're right, of course. I'll fix this. $\endgroup$
    – skd
    Jun 3, 2018 at 16:02
  • $\begingroup$ Here is a failed attempt at answering this question. Suppose $E\wedge I \neq 0$. Then the same argument as in Strickland's answer, combined with the self-duality of the generalized Moore spectra, shows that $[E, M(i_0, \cdots, i_n)] \neq 0$. If $E$ had a finite acyclic $F$, then $[E,F] = 0$. If $F$ has type $N$, then every type $N$ spectrum (in particular, $M(i_0, \cdots, i_N)$) can be obtained from $F$ via a finite number of retracts and cofiber sequences, so we would run into a contradiction. Therefore $E$ cannot have a finite acyclic. $\endgroup$
    – skd
    Jun 4, 2018 at 5:59
  • $\begingroup$ Note that if such an $E$ with a finite acyclic existed, it would be a counterexample. Indeed, $\langle E \rangle \geq \langle X \rangle \geq \langle T(n) \rangle$ for every $n\geq N$, where $X$ has type $N$. As $\langle T(n) \rangle \geq \langle K(n) \rangle$, this implies that $\langle E \rangle \geq \langle K(n) \rangle$ for every $n\geq N$. If $E$ had a finite acyclic $F$, then $K(n)_\ast F=0$ for every $n$, i.e., $F$ would be contractible (as finite spectra are harmonic). $\endgroup$
    – skd
    Jun 4, 2018 at 5:59
  • $\begingroup$ In fact, since $\bigvee_n \langle K(n) \rangle \geq \langle I \rangle$ (all finite spectra are harmonic), this shows that $\langle E \rangle\geq \langle I \rangle$ on finite spectra. I think that this is true on all spectra, and in fact, I believe that the question is equivalent to the following: if $E$ is a spectrum with no finite acyclics, then $\langle E\rangle \geq \langle I\rangle$. (Note that $\langle E\rangle \geq \langle I\rangle$ implies that $E$ has a finite local, namely $S/p$.) I don't know how to prove this equivalence, though. $\endgroup$
    – skd
    Jun 4, 2018 at 6:13

2 Answers 2

12
$\begingroup$

Your question appears to be equivalent to the 'dichotomy conjecture' of Hovey, which I believe is still open.

First, note that any finite spectrum has a type, and all finite spectrum of type $n$ have the same Bousfield class, usually denoted $F(n)$. In Hovey and Strickland's memoir (Appendix B) they conjecture that if $E \wedge I \ne 0$, then $\langle E \rangle \ge \langle F(n) \rangle$ for some $n$. This is precisely your question.

In his paper on the chromatic splitting conjecture, Hovey conjectured that every spectrum has either a finite acyclic or a finite local (the dichotomy conjecture). With Palmieri (The structure of the Bousfield lattice) he proved that the following conjectures are equivalent:

(1) If $E \wedge I \ne 0$, then $\langle E \rangle \ge \langle F(n) \rangle$ for some $n$.

(2) The dichotomy conjecture.

(3) If $E$ has no finite acyclics, then $\langle E \rangle \ge \langle I \rangle$.

The analog of the dichotomy conjecture is known to hold in other categories with a good theory of support satisfying the tensor product property - see 'The Bousfield lattice of a triangulated category and stratification' by Iyengar and Krause.

$\endgroup$
3
  • $\begingroup$ I just noticed your comments to the question - it seems you have already noticed that (1) is equivalent to (3)! $\endgroup$
    – Drew Heard
    Jun 4, 2018 at 8:44
  • $\begingroup$ Awesome, thanks for the references! This means that the question is probably really hard to solve; (3) is an infinite height analogue of the telescope conjecture. Indeed, one equivalent way to rephrase the telescope conjecture is that if the thick subcategory of $E$-acyclics consists of finite spectra of type $n+1$, then $\langle E \rangle \geq \langle K(n) \rangle$. Moreover, $\langle I \rangle \leq \bigvee_n \langle K(n) \rangle$, and I think this is the only class which is less than or equal to $\bigvee_n \langle K(n) \rangle$. $\endgroup$
    – skd
    Jun 4, 2018 at 12:38
  • $\begingroup$ The last part of the last sentence in my comment above is not exactly correct: one also has $\langle I_\mathbf{Z} \rangle \leq \bigvee_n \langle K(n) \rangle$. $\endgroup$
    – skd
    Jun 4, 2018 at 23:06
10
$\begingroup$

I don't think that the answer is known. However, here are some comments. I will work everywhere with $p$-local spectra, for some fixed prime $p$, and write $I$ for the $p$-local Brown-Comenetz spectrum, so that $[W,I]=\text{Hom}(\pi_0(W),\mathbb{Q}/\mathbb{Z}_{(p)})$.

I first claim that $I\wedge W=0$ iff $[W,S_p]_*=0$ (where $S_p$ is the $p$-adic completion of the $0$-sphere spectrum). Indeed, we have $I\wedge W=0$ iff $\pi_*(I\wedge W)=0$ iff $\text{Hom}(\pi_*(I\wedge W),\mathbb{Q}/\mathbb{Z}_{(p)})=0$ iff $[W\wedge I,I]_*=0$ iff $[W,F(I,I)]_*=0$. Moreover, the evident map $S\to F(I,I)$ extends uniquely to give a map $S_p\to F(I,I)$, and one can check that this is an equivalence. The claim follows from this.

In most of the cases that you have quoted where $I\wedge W=0$, the proof actually uses the Adams Spectral Sequence to show that $[W,S_p]_*=0$, and then deduces $I\wedge W=0$ by the argument given above.

Next, you have mentioned that $I\wedge X(2)=0$. Here $X(2)$ is a ring spectrum, and it follows that $I\wedge M=0$ whenever $M$ is an $X(2)$-module. By construction, there is a ring map $X(2)\to MU$, so any $MU$-module is an $X(2)$-module, and so annihilates $I$. In particular we have $E\wedge I=0$ whenever $E$ is one of the standard complex-orientable spectra, such as $BP$, $E(n)$, $K(n)$, $P(n)$, $B(n)$ and so on. Also, as $L_nS$ has the same Bousfield class as $E(n)$, we see that $L_nS\wedge I=0$.

Next, recall that $L_n^fS$ denotes the finite localisation of $S$ of chromatic height $n$. I claim that $L_n^fS\wedge I$ is also zero. If $n=0$ then $L_n^fS=H\mathbb{Q}$, and this is complex-orientable, so we have already seen that $L_n^fS\wedge I=0$. Suppose instead that $n>0$. One can reduce in a standard way to the claim that $(v^{-1}X)\wedge I=0$ whenever $X$ is a finite spectrum of height $n$, and $v$ is a $v_n$-self map of $x$. Equivalently, we must show that $[\Sigma^m v^{-1}X,S_p]=0$ for all $m$. If $|v|=d>0$ then $v^{-1}X$ is the homotopy colimit of the spectra $\Sigma^{-id}X$, and for fixed $m$ we have $[\Sigma^{m-id}X,S_p]=0$ for large $i$ by a connectivity argument. The claim follows from this by the Milnor exact sequence.

I'll now explain one example where I am not sure whether $I\wedge W=0$. Take $F(0)=S^0$, then define $F(n+1)$ recursively to be the cofibre of some $v_n$-self map of $F(n)$, inducing multiplication by $v_n^{i_n}$ in $BP$-theory say. There are then evident maps $F(n)\to F(n+1)$, so we can let $F(\infty)$ denote the homotopy colimit. There is an Adams-Novikov spectral sequence $$ \text{Ext}^{**}(BP_*F(\infty),BP_*) \Longrightarrow [F(\infty),S_p]_*. $$ I suspect that the $E_2$ page is already zero, so $[F(\infty),S_p]_*=0$, so $I\wedge F(\infty)=0$. However, I have not proved this.

UPDATE: Tobias Barthel suggested to me the following proof that $I\wedge F(\infty)=0$ (and that $I\wedge W=0$ for yet another large collection of spectra $W$). Put $K(\mathbb{N})=\bigvee_{n\in\mathbb{N}}K(n)$, and recall that $K(\mathbb{N})$-local spectra are called harmonic, whereas $K(\mathbb{N})$-acyclic spectra are called dissonant. It is a theorem of Hopkins and Ravenel that all suspension spectra are harmonic. We can construct $S/p^n$ as the desuspension of a Moore space, so it is harmonic. We can construct $S_p$ as the homotopy inverse limit of the spectra $S/p^n$, so it is also harmonic. This means that whenever $W$ is dissonant we have $[W,S_p]_*=0$ and so $W\wedge I=0$. In particular, it is not hard to see that $K(\mathbb{N})_*(F(\infty))=0$ and so $F(\infty)\wedge I=0$.

$\endgroup$
2
  • 1
    $\begingroup$ Thanks! Just want to point out one other way to show that $L^f_n I = 0$. Since $L^f_n$-localization is smashing and $\langle L^f_n S \rangle \leq \langle T(0) \vee \cdots \vee T(n)\rangle$, it suffices to show that $T(n) \wedge I = 0$ for all $n$. As $\langle \mathbf{F}_p\rangle \geq \langle I\rangle$ (since the homotopy groups of $I$ are bounded above and torsion), it suffices to show that $T(n)\wedge \mathbf{F}_p = 0$. For $n=0$ this is obvious, and for $n>0$ this follows since $v_n$-self maps have positive Adams filtration. $\endgroup$
    – skd
    Jun 4, 2018 at 2:27
  • $\begingroup$ Regarding the last paragraph: there is a trigraded spectral sequence converging to $\mathrm{Ext}^{s+n,t}(BP_\ast F(\infty), BP_\ast)$ whose $E_2$-page is $\varprojlim^n \mathrm{Ext}^{s,t}(BP_\ast F(m), BP_\ast)$. I don't know if it's any easier to show that these groups vanish. $\endgroup$
    – skd
    Jun 4, 2018 at 2:30

Your Answer

By clicking “Post Your Answer”, you agree to our terms of service and acknowledge that you have read and understand our privacy policy and code of conduct.

Not the answer you're looking for? Browse other questions tagged or ask your own question.